NURS620 QUIZ 1 HEART

Pataasin ang iyong marka sa homework at exams ngayon gamit ang Quizwiz!

What is the most important question to ask when a client presents with chest pain? A. " What were you doing at the time of onset of chest pain? " B. " What was the time of the onset of pain? " C. " Are you a smoker? " D. " When was the last time you had blood work done? "

" What was the time of the onset of pain? " Answer B If your client presents with chest pain, be sure to determine the time of onset because this may influence the course of treatment. Thrombolytic agents must be given within 6 hours of onset of pain to be effective.

Which would be the LAST step you would consider in a client with long-term chronic ischemic heart disease? A. Use of aspirin B. Use of beta blockers, calcium channel blockers, and nitrates C. Risk factor and lifestyle modification D. A coronary angiogram

A coronary angiogram Answer D Long-term secondary prevention for chronic ischemic heart disease includes the use of aspirin, beta blockers, nitrates, and risk factor and lifestyle modification. Further pharmaceutical management may include diuretics, ACE inhibitors and/or angiotensin receptor blockers, spironolactone, and digoxin. Calcium channel blockers for the most part should be avoided. If utilized, the preferred calcium channel blockers include amlodipine or felodipine. To evaluate the progression of the disease, periodic testing should include noninvasive procedures such as exercise stress, stress-echoes, as well as close monitoring of pro-BNP and renal status. Comprehensive lab review likewise includes evaluation of LFTs, TSH, and HgbA1c. Invasive testing, including coronary angiography, should be reserved for clients who have markedly positive stress tests or angina that is refractory to medical treatment.

Which drugs are used to lower blood pressure in a client with coexisting benign prostatic hypertrophy? A. Beta blockers B. ACE inhibitors C. Alpha-adrenergic blockers D. Calcium channel blockers

Alpha-adrenergic blockers Answer C Alpha-adrenergic blockers are used both to lower blood pressure and to relieve some of the symptoms associated with benign prostatic hypertrophy. They decrease the resistance along the prostatic urethra by relaxing the smooth muscle component of the prostate. Alpha-adrenergic blockers can cause postural hypotension during the initial doses of therapy; therefore, the dosage should " start low and go slow " and then gradually increase depending on the effectiveness. Clients should be advised to take an alpha blocker at bedtime because of the profound orthostasis associated with it. Further side effects may include dry mouth and sedation. Clients should be advised of the risk of increased sedation when combined with any sedating medication. The Joint National Committee suggests an alpha blocker should never be instituted as initial solo therapy but rather be added to the most appropriate antihypertensive therapy for overall individual client physiologic comorbidities.

Which drug may be used to convert atrial fibrillation to sinus rhythm? A. Digitalis B. Lidocaine C. Amiodarone (Cordarone) D. Adenosine (Adenocard)

Amiodarone (Cordarone) Answer C Drugs that convert atrial fibrillation to sinus rhythm are amiodarone (Cordarone), disopyramide (Norpace), flecainide (Tambocor), propafenone (Rythmol), and ibutilide (Corvert). Digoxin is effective only for rate control at best and should be used only as a second-line agent. Lidocaine is used for ventricular dysrhythmias, and adenosine (Adenocard) is the treatment of choice in the case of paroxysmal supraventricular tachycardia (PSVT). Adenocard can serve as a diagnostic tool in PSVT, allowing the provider to assess the electrocardiographic presentation by slowing ventricular response, or it may convert the PSVT to normal sinus rhythm (NSR) contingent on the etiology of the electrical conduction disturbance.

The prevalence of which of the following conditions is higher in women, and therefore aggressive teaching needs to be done at every visit? A. Myocardial infarction B. Angina pectoris C. Myocardial ischemia D. Transient ischemic attacks (TIAs)

Angina pectoris Answer B The prevalence of angina pectoris is higher in women than in men, but the prevalence of myocardial infarction is higher in men. Although not always recognized by female clients, coronary heart disease (CHD) still remains the single-most common cause of death in women 20 years or older in the United States

Marvin, age 56, is a smoker with diabetes mellitus. He has just been diagnosed as hypertensive. Which of the following drugs has the potential to cause the development of bronchial asthma and inhibit gluconeogenesis? A. Angiotensin-converting enzyme (ACE) inhibitors B. Beta blockers C. Calcium channel blockers D. Diuretics

Beta blockers Answer B Beta blockers should not be ordered for Marvin for two reasons: He smokes and he is a diabetic. The adverse effects of all beta blockers include the potential for developing bronchial asthma and for inhibiting gluconeogenesis, and therefore they may prolong hypoglycemic episodes.

There are four classifications of heart failure that often are interwoven; they include systolic, diastolic, acute, and/or chronic. Clients who present with JVD, dyspnea with exertion, peripheral edema and abdominal fullness would most likely be experiencing chronic right sided heart failure. A routine diagnostic work up would include all of the following except: A. BNP. B. BMP. C. echocardiogram. D. CTA.

CTA Answer D Diagnostics would include a serum pro-BNP to assess degree of cardiac dysfunction. BNP is a hormone released from the ventricle in response to increased heart workload. It serves as a counterregulatory hormone to counteract innate compensatory mechanism. A BMP would be assessed to determine fluid and electrolyte status as well as renal status. Echocardiography is routinely obtained to monitor myocardial contractility, as well as valvular status. A CTA would not be routinely monitored.

When indicated, which procedure needs endocarditis prophylaxis? A. Vaginal or cesarean deliveries B. Insertion or removal of intrauterine devices C. Dental procedures or extractions D. Body piercings

Dental procedures or extractions Answer C Dental procedures and extractions require antibiotic prophylaxis against bacterial endocarditis when comorbid conditions exist, such as a prior history of a client who has undergone valvular replacement. The recommendations from cardiologists reflect an analysis of relevant literature regarding procedurerelated endocarditis. The recommendations serve only as guidelines. Because endocarditis may occur even with appropriate use of antimicrobials, one must be concerned if fever, night chills, weakness, myalgia, or malaise are reported after procedures.

11-96 Which of the following cardiac tests is used to identify intermittent ectopy and to match symptoms with underlying rhythms and should be used if the interval between symptom occurrence is greater than 48 hours? A. 12-lead ECG B. Holter monitor C. Event monitor D. Echocardiogram

Event monitor Answer C An event monitor is used to identify intermittent ectopy and match symptoms with underlying rhythm and should be used if the interval between symptom occurrence is greater than 48 hours. A 12-lead ECG should be done to evaluate conduction delays, cardiac intervals, and pre-excitation syndromes. A Holter monitor (24 hours) should be ordered to identify frequency of ectopy, evaluate diurnal variations, and match symptoms with underlying rhythm. An echocardiogram should be ordered to evaluate valvular abnormalities and ventricular function.

What are the recommendations when treating high triglycerides? A. Aggressively treat with fenofibrates. B. Treatment should be with combination therapy with a statin and fenofibrate. C. Focus on lifestyle changes as initial therapy. D. Use a bile acid sequestrant.

Focus on lifestyle changes as initial therapy. Answer C Based on an algorithm for screening and managing clients with elevated triglycerides, experts have established an optimal level of triglycerides of less than 100 mg/dL and recommend intensive diet and lifestyle changes for clients with borderline elevated triglycerides. For clients with borderline triglyceride levels of 150 - 199 mg/dL, experts recommend losing 5% of current body weight and limiting carbohydrates to 50% - 60% of daily caloric intake. Weight loss has a beneficial effect on lipids and lipoproteins with a 5% - 10% weight reduction resulting in a 20% decrease in triglycerides, an approximate 15% reduction in LDL cholesterol, and an 8% - 10% increase in HDL cholesterol. The American Heart Association recommends limiting added sugars to less than 10% of daily caloric intake and provides guidance on fructose consumption, recommending that borderline clients consume less than 100 grams. For clients with very high triglyceride levels, drug therapy is used to reduce the risk of pancreatitis. Guidelines suggest using nonfasting triglyceride levels, which simplifies testing for clients and clinicians by eliminating the traditional 12-hour fast. In the United States, 31% of adults have triglyceride levels that exceed 150 mg/dL, and Mexican Americans (36%) are more likely than whites (33%) and blacks (16%) to have high triglycerides.

What is a causative factor in the formation of blood clots causing deep venous thrombosis? A. Blood flow turbulence B. Intact vessels C. Thin blood D. Hypercoagulability

Hypercoagulability Answer D The causative factors in the formation of blood clots are referred to as Virchow ' s triad. They include venous stasis, vessel injury, and hypercoagulability.

Which resuscitation recommendation was made by the American Heart Association to improve survival rates in clients with a return of spontaneous circulation (ROSC) following a cardiac arrest? A. Hypothermia B. Hyperthermia C. Initiation of intravenous lidocaine D. A rebreathing mask

Hypothermia Answer A Resuscitation Guidelines 2010 from the American Heart Association recommend, for the post - return of spontaneous circulation (ROSC) to the client who remains nonresponsive, that mild hypothermia should be provided to improve survival rates. Studies suggest the mortality rate in clients provided cooling was lower, and there was also an improvement in neurological outcomes. For clients who remain nonresponsive post-ROSC with no contraindications, rapid cooling to a core temperature of 32 - 34 ° C should be initiated. The client should be maintained at this temperature for 12 - 24 hours at which time slow reversal of the cool state should be initiated. Further recommendations address the management of hemodynamics, electrocardiographics, fluid and electrolytes, and a purposeful assessment to determine the etiology to optimize the outcome.

Which of the following is usually the earliest sign or symptom of chronic occlusive arterial disease in the extremities? A. Loss of hair over the lower extremity B. Intermittent claudication C. Painful ulcerations of the toes of the affected extremity D. Muscle atrophy

Intermittent claudication Answer B The clinical symptoms of chronic occlusive arterial disease occur slowly over a period of years and result primarily from tissue underperfusion and ischemia. Usually, the earliest symptom is intermittent claudication. Later signs and symptoms of chronic occlusive arterial disease in the lower extremities include changes in skin texture, loss of hair, muscle wasting, reduced muscle strength and sensation, and the development of ulcers on the toes or heels.

Which of the following statements is true concerning auscultation of the typical murmur associated with aortic stenosis? A. It is a harsh, crescendo-decrescendo ejection type that often radiates to the carotid arteries. B. It is a diastolic murmur. C. It is best heard at the apex of the heart. D. The loudness of the murmur reflects the severity of the lesion.

It is a harsh, crescendo-decrescendo ejection type that often radiates to the carotid arteries. Answer A Auscultation of the typical murmur associated with aortic stenosis usually reveals that it is a systolic murmur of a harsh, crescendo-decrescendo ejection type, best heard at the base of the heart. It often radiates to the carotid arteries. In clients with a calcified aortic valve, the murmur is harsh and rasping at the base. As the client ' s left ventricular dysfunction worsens, the typical harsh murmur becomes quieter and resembles the murmur of mitral regurgitation, which often is a secondary diagnosis due to left ventricular dysfunction. The loudness of the murmur does not reflect the severity of the lesion.

Which of the following statements is true of auscultation of the aortic valve? A. It is best performed using the diaphragm of the stethoscope. B. It is best heard at the second left intercostal space. C. Auscultation need only be done at one site. D. It is best performed using the bell of the stethoscope.

It is best performed using the diaphragm of the stethoscope. Answer A Auscultation of the aortic valve (the second heart sound [S 2 ], which signifies closure of the aortic and pulmonic valves) is best performed using the diaphragm of the stethoscope because it is a high-pitched sound and is best heard at the second right intercostal space. It should also include auscultation at other locations such as neck, apex, and right parasternal region because important findings are sometimes present in other locations.

An anterior wall myocardial infarction most likely occurs from occlusion of the A. left circumflex artery. B. left main artery. C. right coronary artery. D. left anterior descending artery.

Left anterior descending artery Answer D Occlusion of the left anterior descending artery results in damage to the anterior septal wall and the left ventricle. Occlusion of the right coronary artery usually results in damage to the inferior wall of the left ventricle, as well as potential damage to the right atrium, right ventricular, SA node, AV node, and papillary muscles of the mitral valve. When the left circumflex artery (LCX) is the dominant vessel, occlusion can be responsible for damage to the inferior wall of the left ventricle. The LCX normally supplies the lateral wall of the left ventricle, as well as the posterior wall.

You are managing a client with CHF. The client is presently on Carvedilol 12.5 mg bid, Lisinopril 2.5 mg PO daily, Lasix 80 mg PO daily, and Spironolactone 25 mg PO daily. He is in to see you for a 2-week follow-up and shares he has gained 7 lbs in the last week. He shares he has been faithful to his fluid and dietary restrictions and that his urinary output is somewhat less than it has been. What medication could you add to optimize the response to the loop diuretic? A. Bumex B. Metolazone C. Demadex D. Diamox

Metolazone Answer B The addition of metolazone is the appropriate next intervention in the management of congestive heart failure that presents with a decreased sensitivity to loop diuretics and Aldactone combination. The client should be reminded of the importance of strict medication and dietary recommendations. He should likewise be counseled to contact the office if his weight increases by 3 lbs or more in 24 hours or 5 lbs in 1 week. With the addition of metolazone, the client should be reminded of the importance of maintaining a home blood pressure log. In addition, he should contact the office if increasing shortness of breath or dyspnea on exertion occurs or if he experiences abdominal distension, palpitations or syncopal events.

Which of the following causes coronary valve leaflets to billow into the atrium during ventricular systole and runs in families? A. Mitral regurgitation B. Mitral valve prolapse C. Aortic regurgitation D. Aortic stenosis

Mitral valve prolapse Answer B Mitral valve prolapse causes one or both leaflets to billow into the atrium during ventricular systole and runs in families.

Which of the following statements is true about hypertension (HTN) during pregnancy? A. Methyldopa (Aldomet) is the drug of first choice for control of mild to moderate HTN in pregnancy. B. Beta blockers are safe during the pregnancy. C. ACE inhibitors (I and II) are safe during all trimesters. D. Beta blockers are safe only in the first trimester of pregnancy.

Methyldopa (Aldomet) is the drug of first choice for control of mild to moderate HTN in pregnancy. Answer A Evidence-based guidelines from the American Association of Clinical Endocrinologists single out methyldopa and nifedipine as preferable antihypertensive medications in pregnancy, with magnesium sulfate for women with pre-eclampsia who are at high risk for seizures. Methyldopa is the drug of first choice for control of mild to moderate hypertension in pregnancy and is the most widely prescribed antihypertensive for this indication. The wide use of this drug in pregnancy reflects the fact that it has the best documented maternal and fetal safety record, including favorable long-term (4.5 - 7.5 year) pediatric follow-up data. During longterm use in pregnancy, methyldopa does not alter maternal cardiac output or blood flow to the uterus or kidneys, and for all these reasons, it is generally considered the agent of choice for chronic blood pressure control in pregnancy. While earlier studies suggested that administration of beta blockers (particularly those without intrinsic sympathomimetic activity) during pregnancy might increase the chance of intrauterine growth retardation, recent studies have been more reassuring on this point. Nevertheless, the available data are insufficient to rule out unrecognized adverse effects of early and prolonged use of beta blockers pregnancy. When used for short periods (less than 6 weeks) during the third trimester, beta blockers are effective and well tolerated provided there are no signs of intrauterine growth impairment. ACE inhibitors and angiotensin receptor blockers (ARBs) have been associated with serious fetal abnormalities.

What is a common funduscopic change associated with hypertension? A. Optic disk swelling B. Gray lesions C. Deep intraretinal hemorrhages D. A cup-disk ratio greater than 1:2

Optic disk swelling Answer A Funduscopic changes associated with hypertension include optic disk swelling, AV nicking, hard exudates, creamy yellow lesions, and soft exudates such as cotton wool (also seen with subacute bacterial endocarditis). Deep intraretinal hemorrhages (dot hemorrhages) are seen with diabetes. A cup-disk ratio greater than 1:2 is seen with open-angle glaucoma.

Impaired blood flow to the extremities is caused by which of the following common disorders? A. Raynaud ' s disease B. Peripheral vascular disease C. Polycythemia D. Buerger ' s disease

Peripheral Vascular Disease Answer B Impaired blood flow to the extremities results in leg aches and is most often caused by peripheral vascular disorders. Peripheral vascular disease (PVD) affects the arteries and veins. When the disease is arterial, it is usually the result of accumulated fatty streaks and fibrous plaques (high levels of low-density lipoproteins). When the disease is venous, problems relate to venous incompetence secondary to valve obstruction, leading to chronic venous insufficiency and varicose veins. All of the other conditions listed are appropriate differential diagnoses, and a thorough history and physical examination must be performed to rule them out. However, the most common cause of leg aches remains peripheral vascular disorders.

Which of the following is usually indicative of an abdominal aortic aneurysm? A. RUQ tenderness B. Venous hum in abdomen C. Ascites D. Positive bruit or wide, diffuse pulsation in epigastric area

Positive bruit or wide, diffuse pulsation in epigastric area Answer D A positive bruit or wide, diffuse pulsation in the epigastric area is indicative of an abdominal aortic aneurysm. Right upper quadrant tenderness, a venous hum, and ascites are indicative of right-sided congestive heart failure

A client presents with substernal chest pain that is provoked by exertion and relieved by rest and nitroglycerin. What do you suspect? A. Stable angina B. Unstable angina C. Acute coronary syndrome (ACS) D. Simple overexertion

Stable angina Answer A Substernal chest pain that is provoked by exertion and relieved by rest or nitroglycerin is considered typical or stable angina and carries a high risk for unstable coronary artery disease (CAD). Chest pain radiating to one or both arms, sometimes accompanied by nausea, vomiting, or diaphoresis, is associated with a high risk of acute coronary syndrome. " Anginal equivalents " may include symptoms not typically associated with CAD, such as discomfort radiating to the jaw, neck, or ear, if they are also associated with exertion. New-onset or worsening exertional dyspnea is the most common anginal equivalent in the elderly.

Dana has ischemic arterial ulcers. What is your first priority when counseling her? A. Tell her that increasing coffee intake will stimulate heart rate and circulation. B. Tell her to decrease water intake slightly to improve blood viscosity. C. Tell her to reduce risk factors to improve tissue perfusion. D. Tell her to begin an intense aerobic program.

Tell her to reduce risk factors to improve tissue perfusion. Answer C The first priority for managing ischemic arterial ulcers is to improve tissue perfusion. When counseling clients, tell them to decrease their risk of further damage by reducing risk factors. Ways to reduce risk factors include stopping smoking, reducing fat (lipid) intake, avoiding restrictive garments and trauma (mechanical, chemical, and thermal), maintaining adequate hydration, and avoiding caffeinated drinks. Smoking and caffeine can cause vasoconstriction and reduce blood flow. Lipid lowering can restore some of the arterial vessels ' ability to vasodilate and " self-regulate " blood flow in response to tissue needs. Adequate hydration reduces blood viscosity, improves blood flow, and reduces chances for microemboli formation. Although an intense aerobics program should not be initiated, Buerger-Allen exercises should be done three to four times per day. The client raises the extremity 45 ° and then lowers it to a supine position, repeating this five times over a 2-minute period. The changes in position cause the arteries of the legs to refill by gravity.

Which of the following statements about hypertension is true? A. It is frequently caused by pheochromocytoma. B. It is usually the result of an underlying, correctable problem. C. The cause is unknown in approximately 95% of cases. D. It has a higher incidence among adult white men than any other group.

The cause is unknown in approximately 95% of cases. Answer C The cause of hypertension is unknown (idiopathic) in approximately 95% of cases. This form of hypertension is known as primary or essential hypertension. The remaining 5% of cases are secondary to underlying disease processes such as renal disease, renal artery stenosis (RAS), pregnancy, endocrine disorders, pheochromocytoma. Confounding issues to address include obesity, NSAID usage, alcohol usage, and smoking. The incidence of hypertension is greatest in adult black men and women.

Harry comes to your office with waxing and waning ischemic symptoms over a period of days and weeks, an increase in angina while at rest and transient ST changes on his electrocardiogram. This presentation leads you to believe that he is experiencing: A. a stroke. B. a myocardial infarction. C. stable angina. D. unstable angina.

Unstable Angina Answer D The clinical presentation of unstable angina may include waxing and waning of ischemic symptoms over a period of days or weeks. It often involves a progressive increase in symptoms in those with previous stable angina, including rest angina, and may include transient ST changes on the electrocardiogram. The differential diagnosis of unstable angina versus ST-segment elevation myocardial infarction (STEMI) versus non- STEMI is confirmed by the elevation of serum troponin. Important to note is the possibility of acute coronary syndrome without the alteration in electrocardiographic display.

The most common cause of elevated total and low-density lipoprotein cholesterol levels in the United States is A. heredity. B. hypothyroidism. C. diabetes. D. a diet high in saturated fat.

a diet high in saturated fat. Answer D Although heredity, hypothyroidism, and diabetes contribute to abnormal serum lipid levels, a diet high in saturated fat is the most common cause of elevated total and low-density lipoprotein cholesterol in the United States.

Terry, a 42-year-old black man who just moved into the area, comes into the clinic for a new-client visit. He brings his medical records from his previous health-care provider; the records show a blood pressure of 140/104 mm Hg on two separate occasions. Recent laboratory tests (complete blood count, chemistry profile, urinalysis, and thyroid-stimulating hormone) are normal. A recent electrocardiogram shows normal sinus rhythm with left ventricular hypertrophy. He denies any medical problems and tells you he has never been diagnosed with hypertension. He is not taking any medications, does not smoke, and drinks about two beers a day. He is currently unemployed. His blood pressure today is 150/110 mm Hg. Your next step would be to A. obtain plasma and urine catecholamine measurements. B. have him keep a food diary for 1 week, then return for a repeat blood pressure reading. C. begin drug therapy with hydrochlorothiazide (HydroDIURIL) 25 mg. D. start him on metoprolol (Lopressor) 100 mg twice a day.

begin drug therapy with hydrochlorothiazide (HydroDIURIL) 25 mg. Answer C Pharmacological therapy is indicated for Terry because his blood pressure has been elevated on three separate occasions. Target organ damage to the heart is suggested by the left ventricular hypertrophy noted on his electrocardiogram, which may suggest the need for a two-drug combination. Diuretics such as hydrochlorothiazide (HydroDIURIL) have been proved to reduce hypertensive morbidity and mortality and should be the first drug of choice. In addition to being inexpensive, diuretics, when used as antihypertensive agents in blacks who have low plasma renin activity, are more potent than other antihypertensive drugs. This client may benefit from a second line addition of an ACE inhibitor, an angiotensin receptor blocker, a beta blocker, or a calcium channel blocker. The client does not report symptoms suggesting pheochromocytoma, and its occurrence is rare, so plasma and urine catecholamine level tests are not indicated at this time. Analyzing the client ' s food diary, although a necessary intervention, will not sufficiently meet this client ' s needs as priority intervention.

Liver function tests should be monitored routinely every 4 months in the client on maintenance therapy with all hypolipidemic drugs except: A. bile acid sequestrants (Questran, Colestid). B. hydroxymethylglutaryl-coenzyme A reductase inhibitors (statins). C. nicotinic acid (niacin). D. fibric acid derivatives (gemfibrozil).

bile acid sequestrants (Questran, Colestid) Bile acid sequestrants (Questran, Colestid) are not absorbed from the gastrointestinal tract and lack systemic toxicity; therefore, liver function tests do not need to be monitored in clients taking these drugs. Hepatotoxicity is a serious, yet infrequent, adverse effect of hydroxymethylglutaryl-coenzyme A reductase inhibitors (statins), occurring about 1% of the time. Hepatic enzymes should be assessed before initiation and then as indicated secondary to dosage adjustment and/or product labeling. Nicotinic acid (niacin) is contraindicated in clients with active liver disease, peptic ulcer disease and or severe gout. Fibric acid derivatives such as gemfibrozil are contraindicated in clients with severe renal or hepatic disease.

The leading cause of death in women in the United States is A. trauma. B. cardiovascular disease. C. diabetes. D. cancer.

cardiovascular disease. Answer B The leading cause of death for women in the United States is cardiovascular disease, including heart and cerebrovascular disease. Although the incidence of cardiovascular disease is lower in younger women, it dramatically increases in women older than age 45 and is a major cause of chronic illness and disability in these women. Women often present atypically and subsequently have higher morbidity and mortality rates related to CAD secondary to lost opportunity to treat because the diagnosis of acute coronary syndrome is often missed.

Mitral valve prolapse is characterized by A. elongation of the chordae tendineae and enlarged valve leaflets. B. ballooning (prolapse) of the cusps into the ventricle during diastole. C. an early diastolic murmur. D. an early systolic murmur.

elongation of the chordae tendineae and enlarged valve leaflets. Answer A Mitral valve prolapse (MVP) is a congenital syndrome characterized by elongation of the chordae tendineae and enlarged valve leaflets. Ballooning of the cusps (prolapse) into the atrium occurs to varying degrees during ventricular systole. MVP is characterized by a midsystolic click and late systolic murmur.

Many clients with mitral valve prolapse exhibit A. a slow heart rate. B. somnolence. C. a lengthened PR interval on electrocardiogram. D. fatigue.

fatigue. Answer D Although many clients with mitral valve prolapse (MVP) are asymptomatic, clients may appear with a variety of symptoms, including what some have described as " MVP syndrome " : a characteristic click, fatigue, palpitations, postural hypotension, chest pain, atrial and ventricular arrhythmias, anxiety, and symptoms of autonomic dysfunction (excessive secretion of catecholamines causing vasoconstriction and orthostatic tachycardia)

Deep vein thrombosis may result in A. generalized edema of the involved extremity. B. atrophy of leg muscles. C. loss of sensation in the affected extremity. D. the release of fat emboli.

generalized edema of the involved extremity. Answer A Deep vein thrombosis (DVT) may result in generalized edema of the involved extremity. DVT usually originates in the pelvis and lower extremities. It is typically asymptomatic. Pulmonary embolus may be the first indication of thrombosis. Thrombus formation may not be clinically apparent because of the large capacity of the venous system and development of collateral circulation around obstructions. Pain is the most common symptom and may be aching or throbbing. Most pulmonary emboli are caused by DVT. There is no atrophy of the leg muscles, and although there is pain, there is not a loss of sensation. DVT may result in a thromboembolism. Other emboli include tumors that have invaded the venous circulation (tumor emboli), amniotic fluid, air, fat, bone marrow, or foreign intravenous material.

Jamie, age 49, who has a history of hyperlipidemia, has symptoms that lead you to suspect unstable angina. Your next action would be to A. start aspirin therapy and schedule an exercise stress test at the client ' s convenience. B. initiate lipid-lowering agents. C. hospitalize the client in a monitored setting with pharmacological control of ischemia, arrhythmias, and thrombosis as appropriate. D. prescribe a Holter monitor and start her on a beta blocker.

hospitalize the client in a monitored setting with pharmacological control of ischemia, arrhythmias, and thrombosis as appropriate. Answer C According to the Agency for Healthcare Research and Quality clinical practice guidelines, clients with angina at rest, postinfarction angina, or rapidly progressive unstable angina with electrocardiogram changes should be managed with hospitalization in a monitored setting, bedrest, control of precipitating factors, and initiation of medical therapy. The American Heart Association in concert with an International Liaison Committee recommends percutaneous coronary intervention (PCI) for timely diagnostic and potential revascularization at the point of care.

To reduce the incidence of coronary events in an individual without coronary artery disease who has two or more risk factors, the goal serum low-density lipoprotein cholesterol level should be A. 170 - 190 mg/dL. B. 150 - 170 mg/dL. C. 130 - 150 mg/dL. D. less than 130 mg/dL.

less than 130 mg/dL Answer D The National Cholesterol Education Program (NCEP) guidelines recommend a goal serum lowdensity lipoprotein cholesterol level of less than 130 mg/dL for individuals without coronary artery disease but with two or more risk factors.

Rona, age 69, has hypertension (HTN), drinks one glass of white wine per day, and is slightly overweight. She asks you if making changes in her life at this age will make any difference. You tell her that lifestyle modifications for the control of HTN A. are not as effective in older adults because HTN is an inevitable consequence of aging. B. require a marked reduction in weight and a very limited choice of foods to achieve any benefit. C. should include at least three glasses of red wine every day because it improves high-density lipoprotein cholesterol levels, a known cardiovascular risk factor, which may be worsened by HTN. D. may prevent HTN, lower elevated blood pressure, and reduce the number and dosage of antihypertensive medications needed to manage a condition.

may prevent HTN, lower elevated blood pressure, and reduce the number and dosage of antihypertensive medications needed to manage a condition. Answer D Lifestyle modifications are essential for the control of hypertension (HTN). Weight reduction of as little as 4.5 kg (10 lb) reduces blood pressure in a large proportion of overweight persons with HTN. It can also enhance the blood pressure - lowering effect of concurrent antihypertensive agents and reduce cardiovascular risk factors. The Dietary Approaches to Stop Hypertension (DASH) recommendations allow a combination of foods, with a diet rich in fruits, vegetables, and low-fat dairy products, and are not based on excessive calorie restriction. Elevated blood pressure is not an inevitable consequence of aging. Excessive alcohol intake is an important risk factor for HTN because it can cause resistance to antihypertensive therapy. Individuals who drink should be counseled to limit their daily intake to no more than 1 oz of ethanol for average-sized men and 0.5 oz of ethanol for women or lighter-weight individuals (1 oz of ethanol equals 2 oz of whiskey, 24 oz of beer, or 10 oz of wine).

Larry, age 66, is a smoker with hypertension and hyperlipidemia. He is 6 months post MI. To prevent reinfarction, the most important behavior change that he can make is to A. quit smoking. B. maintain aggressive hypertension therapy. C. stick to a low-fat, low-sodium diet. D. continue with his exercise program.

quit smoking. Answer A All of these behavior changes are important, but for smokers, quitting is the single-most important change they can make to reduce future risk of MI.

An active 68-year-old man under your care has known acquired valvular aortic stenosis and mitral regurgitation. He also has a history of infectious endocarditis. He has recently been told he needs elective replacement of his aortic valve. When he comes in, you discover that he has 10 remaining teeth in poor repair. Your recommendation would be to: A. defer any further dental work until his valve replacement is completed. B. instruct the client to have dental extraction done cautiously, having no more than two teeth per visit removed. C. suggest that he consult with his oral surgeon about removing all the teeth at once and receiving appropriate antibiotic prophylaxis. D. coordinate with his cardiac and oral surgeons to have the tooth extraction and valve replacement done at the same time to reduce the risk of anesthetic complications.

suggest that he consult with his oral surgeon about removing all the teeth at once and receiving appropriate antibiotic prophylaxis. Answer C The client ' s poor dental status is a probable source of endocarditis. Proper management of his dental work is essential before valve replacement because it will lower the risk for endocarditis. Removing the teeth in a single procedure reduces the repetitive risks of extraction-related bacteremia, as well as the possibility of developing antibiotic resistance from repeating prophylaxis regimens. Performing the extraction concurrently with valve surgery will add stress and risk of bacteremia to the procedure.

At least 75% of clients need two or more antihypertensive agents to reach their blood pressure goal. In choosing combination therapy for a client with stage 2 hypertension (HTN) (SBP greater than 160 and or DBP greater than 100) in a 59-year-old male with a PMH relevant for uncontrolled essential HTN, DM, SOB, DOE, stage 1 diastolic heart failure, with an EF of 55% and a serum creatinine of 0.4, the best initial combination treatment would include A. Lasix and a beta blocker. B. thiazide diuretic and an ACE inhibitor. C. nitrate and a beta blocker. D. thiazide and a calcium channel blocker.

thiazide diuretic and an ACE inhibitor. Answer B In a client with uncontrolled essential hypertension, the risk of the development of Diastolic heart failure is a concern as the increased workload on the heart has the propensity to cause left ventricular hypertrophy (LVH) and subsequent dysfunction. Thiazide diuretic and an ACE inhibitor in the client described in this scenario is the initial combination therapeutic of choice. Thiazide therapy will assist in the achievement of a stable fluid balance, and an ACE inhibitor in the client with diabetes mellitus is well supported as being efficacious. While a beta blocker may be considered, studies suggest the choice of a beta blocker should be specific when instituted in the client with heart failure. Coreg (Carvedilol) and metoprolol succinate (Toprol - XL) are noted to be superior in decreasing morbidity and mortality. These may be considered in this client if exquisite blood pressure management is not achieved and added in a further combination therapeutic plan.

Pharmacological therapy for mitral valve disease includes A. treatment of dyspnea with diuretics to relieve congestion. B. reduction of fast ventricular rates with digoxin, beta blockers, or calcium channel blockers. C. preload reduction with antihypertensive agents to decrease regurgitant flow. D. daily antibiotic use to ward off bacterial infections

treatment of dyspnea with diuretics to relieve congestion. Answer A Pharmacological therapy for mitral valve disease includes treatment of dyspnea with diuretics to relieve congestion. It also consists of afterload (not preload) reduction with antihypertensive agents to decrease regurgitant flow. Other therapies may include antiarrhythmics for atrial fibrillation because this condition can exacerbate symptoms as a result of ventricular filling. Beta blockers may be prescribed for clients with mitral valve prolapse (MVP) to help control palpitations; however, fatigue is a problem with MVP, and beta blockers may exacerbate the fatigue. Antibiotic prophylaxis to prevent infective bacterial endocarditis is not indicated because the client with MVP is at no greater risk of developing endocarditis than the general population. Anticoagulation with acetylsalicylic acid (ASA [aspirin]; 81 - 325 mg/d) is prescribed for some individuals with MVP who also have a history of transient ischemic attack (TIA), ischemic stroke, or atrial fibrillation.

Cough, loss of taste, and rash are adverse effects associated with which class of antihypertensive agents? A. Diuretics B. Beta blockers C. ACE inhibitors D. Calcium channel blockers

ACE inhibitors Answer C A cough, loss of taste, and a rash are all adverse effects of ACE inhibitors. They all disappear with discontinuance of the medication and may or may not reappear if therapy is resumed with another ACE inhibitor. Some diuretics may cause electrolyte disturbances, fatigue, and, depending on the type, rash and impotence. Beta blockers may cause bronchospasm, increase plasma triglyceride levels, and result in central nervous system disturbances. Calcium channel blockers may cause adverse vasodilatory effects such as headache, flushing, palpitation, and ankle edema.

Clients with Prinzmetal ' s angina frequently have a history of Raynaud ' s disease and which other disorder? A. Syncope B. Insomnia C. Migraine headaches D. Leg cramps

Migraine headaches Answer C Prinzmetal ' s (variant) angina usually has no precipitating factors and occurs at rest as a result of spasms of the coronary artery. Clients with Prinzmetal ' s angina frequently have a history of migraine headaches and Raynaud ' s disease.

Janice, age 64, arrives at the office this morning without an appointment. She appears quite anxious and pale and is complaining of an intermittent aching across her sternum and into her jaw and left arm that started about an hour ago and woke her out of a sound sleep. She took an antacid and acetaminophen (Tylenol), but they did not seem to help. Her blood pressure is 160/90 and heart rate is 98. An electrocardiogram shows normal sinus rhythm with 2-mm ST-segment elevations in leads II, III, and AVF. What do you suspect? A. An acute anterior wall myocardial infarction (MI) B. An acute inferior wall MI C. Severe gastrointestinal reflux D. An anxiety attack

An acute inferior wall MI A 12-lead electrocardiogram (ECG) showing 2-mm ST-segment elevations in leads II, III, and AVF indicates an acute inferior wall myocardial infarction (MI). When ST-segment elevation MI (STEMI) in the inferior leads are noted, a right-sided 12- lead ECG should be included because up to 30% of clients with inferior-wall MI have been noted to have right ventricular involvement, which then changes therapeutic management strategies. When a right ventricular involvement is present, the provider should be conservative with the administration of nitroglycerin (NTG) and aggressive fluid offloading. Every attempt to optimize fluid and electrolyte stability should be undertaken, and as in all cases of STEMI, the client should be quickly transported to a percutaneous coronary intervention center for diagnostic work-up. In specific situations intervention may be streamlined. With inferior wall MI, further complications may include disturbances located at the SA node (bradycardia) or AV node (first-degree AV block, second-degree type 1 Mobitz heart block, third-degree heart block), and the health-care provider may auscultate the acute finding of a systolic murmur secondary to papillary muscle involvement of the mitral valve. Ischemic syndromes often follow a circadian rhythm, with an MI occurring more frequently in the morning hours between 8 a.m. and noon. Ischemic heart disease should always be a consideration in female clients because it causes 23% of all deaths in women and is the leading cause of death in women older than age 50. In anterior wall MIs, the danger of left ventricular failure and acute-onset pulmonary edema is always a concern. Once again, exquisite management of fluid and electrolytes is necessary. As it relates to conduction abnormalities, it is not uncommon to note the development of a second-degree type 2 heart block or third-degree heart block. Bundle branch blocks may likewise be noted. In the occurrence of a right bundle branch block, the clinician should always review the axis to assess for the development of a bifascicular block because it may influence therapeutic decisions. Gastrointestinal reflux may produce pain suggestive of angina but is usually related to a heavy meal, occurs in recumbency or bending over, and is usually relieved by antacids, which Janice ' s was not. Anxiety attacks do not usually wake someone out of a sound sleep and would not produce an abnormal ECG except for tachycardia.

Which of the following antihypertensive agents would most likely produce a rebound hypertensive crisis following its abrupt withdrawal? A. Doxazosin (Cardura) B. Lisinopril (Prinivil) C. Losartan (Cozaar) D. Clonidine (Catapres)

Clonidine (Catapres) Answer D Abrupt withdrawal of clonidine (Catapres), a central acting alpha agonist, will most likely produce a rebound hypertensive crisis. Blood pressure may return to or even exceed pretreatment levels after withdrawal of clonidine. This response is rarely seen following withdrawal of beta blockers, diuretics, ACE inhibitors (e.g., lisinopril), or calcium channel blockers. The most noticeable symptoms include sweating, palpitations, nervousness, headache, abdominal cramping, and nausea, along with elevated blood pressure. The mechanism for clonidine withdrawal is thought to be catecholamine overproduction.

Your post-MI client is allergic to ASA. Which drug should he be on indefinitely? A. Coumadin B. Heparin C. Pentoxifylline (Trental) D. Clopidogrel (Plavix)

Clopidogrel (Plavix) Answer D All clients who have sustained a myocardial infarction (MI) and in whom the therapy is not contraindicated should continue to take ASA (acetylsalicylic acid, or aspirin) indefinitely. For clients who are allergic to or intolerant of aspirin, clopidogrel can be substituted. Note the following properties of the oral antiplatelet drugs Brilinta, Plavix, and Effient: Brilinta (ticagrelor) competes with Plavix (clopidogrel) and Effient (prasugrel) for acute coronary syndrome or after placement of a stent. Plavix was the first antiplatelet agent approved by the Food and Drug Administration (FDA) and is used most often, but it has a delayed onset and variable response because it has to be activated by cytochrome P 450 (CYP3A4) enzymes in the liver. Effient is more effective than Plavix but causes more bleeding, so it is not for clients with a prior stroke or transient ischemic attack or those over age 75. Brilinta seems to be more effective than Plavix for acute coronary syndrome and usually does not cause any more major bleeding than Plavix does. For every 1,000 clients with acute coronary syndrome treated for a year, Brilinta prevents 11 more cardiovascular deaths, 11 more MIs, and at least 6 more stent thromboses than does Plavix. But these statistics are affected by the dose of aspirin used with Brilinta. More than 100 mg/d of aspirin makes Brilinta less effective. Ensure that clients on Brilinta take only 81 mg/d of aspirin. Brilinta has a faster onset than Plavix and Effient because it is not a prodrug. It also wears off faster because it binds to platelets reversibly instead of permanently. This fast offset is an advantage for clients who need coronary artery bypass graft or other surgery. But both Brilinta and Plavix should be stopped 5 days before surgery, and Effient should be stopped at least 7 days beforehand. Brilinta's short duration may be a disadvantage in the long run because it has to be taken twice a day instead of once a day like the others. If a client is on Brilinta, avoid using strong CYP3A4 inhibitors or inducers, such as clarithromycin, ketoconazole, and rifampin. Do not exceed 40 mg/d for simvastatin or lovastatin, and monitor digoxin levels when starting or stopping Brilinta. Keep in mind that Brilinta can cause dyspnea, especially during the first week. Consider switching to Plavix or Effient if needed.

Which of the following findings are suggestive of renovascular hypertension? A. Bilateral flank pain on percussion B. Renal arterial bruits in the abdomen, flanks, or back C. A palpable mass in the right lower quadrant D. Decreased urine output

Renal arterial bruits in the abdomen, flanks, or back Answer B Renal artery bruits, which indicate renal artery stenosis, particularly those of diastolic timing, are a suggestive sign of renovascular hypertension. They are often heard or palpated (as a thrill) in the abdomen, flanks, or back. Systolic bruits are commonly detected, especially in older adults, and may not be associated with renal artery stenosis.

During pregnancy, many women develop which cardiovascular change? A. Diastolic murmur B. Hypertension C. Bradycardia D. Systolic murmur

Systolic murmur Answer D Several changes to the cardiovascular system can be seen during pregnancy. The increase in cardiac workload often results in the development of a systolic murmur. A mammary souffle is a murmur that develops from increased blood flow through the mammary artery occurring late in pregnancy or during lactation. As the pregnancy progresses and the diaphragm rises, the heart rises and rotates, displacing the apical impulse. The blood pressure is usually lower during the first and second trimesters, and the heart rate is slightly higher (not slower, as in bradycardia) than in the nonpregnant state.

Sexual activity is a major concern for clients with chronic ischemic heart disease. Which statement is true? A. The sexual partner should be included in the education process. B. The physical stress of sexual intercourse is equivalent to running a half mile. C. Antianginal medication taken just after sexual activity can help prevent symptoms. D. Sexual activity should be attempted only in the morning when the client is well rested.

The sexual partner should be included in the education process. Answer A True statements concerning sexual activity for clients with chronic ischemic heart disease include that the sexual partner should be included in the education process, that antianginal medication taken before sexual activity can help prevent symptoms, and that sexual activity should be attempted when the client is well rested, although morning hours may not always be best. Chronic angina exhibits a circadian rhythm characterized by a propensity toward transient ischemic episodes in the morning hours. The physical stress of sexual intercourse is usually equivalent to that of climbing one flight of stairs at a normal pace or any other activity that induces a heart rate of 120 beats per minute. Clients should be reassured that if they can carry out these activities without symptoms, they can probably engage in sexual activity without symptoms. If symptoms occur, they may be managed with proper precautions such as timing activities more than 2 hours after meals, taking an extra dose of a short-acting beta blocker 1 hour before intercourse, or taking nitroglycerin 15 minutes before intercourse.

Nathan, age 63, comes for his annual physical. He has a history of mild hypertension and hyperlipidemia that he has not been successful in treating by diet and weight loss. His only complaint is a problem with impotence. On physical examination, you note a palpable, pulsatile abdominal mass in the umbilical region; a bruit above the umbilical region; and diminished femoral pulses. You suspect A. renal artery stenosis. B. an abdominal aortic aneurysm. C. a cardiac tumor. D. a thoracic aortic aneurysm.

an abdominal aortic aneurysm. Answer B A pulsating abdominal mass in the region of the umbilicus is suggestive of an abdominal aortic aneurysm. Abdominal aortic aneurysms may be asymptomatic or may cause abdominal or back chapter 11: CARDIOVASCULAR PROBLEMS 279 pain as well as impotence or difficulties with digestion. Abdominal aortic aneurysms can also cause chest pain. Thoracic aneurysms must be quite large to produce symptoms and consequently may be discovered incidentally by chest x-ray. The symptoms are caused by expansion and compression of adjacent thoracic structures and may include dysphagia, hoarseness, and edema of the head and arms. Aneurysm rupture is catastrophic and may present with severe chest, abdominal, or back pain; paralysis; or shock. Renal artery stenosis is a common cause of secondary hypertension and presents in the same manner as essential hypertension. Primary cardiac tumors are rare and are usually atrial myxomas. Metastases to the heart from malignant tumors elsewhere in the body are more frequent. The client usually presents with fever, malaise, weight loss, leukocytosis, an elevated erythrocyte sedimentation rate, and peripheral or pulmonary emboli.

To determine the presence of target organ damage and other risk factors in the client with hypertension, basic diagnostic tests that should be ordered include A.chest x-ray, electrocardiogram, urinalysis, complete blood count, chemistry profile, lipid profile, and thyroid-stimulating hormone (TSH) level. B. renal arteriogram. C. plasma renin activity and 24-hour urinary sodium. D. echocardiogram.

chest x-ray, electrocardiogram, urinalysis, complete blood count, chemistry profile, lipid profile, and thyroid-stimulating hormone (TSH) level. Answer A To determine the presence of target organ damage and other risk factors in the client with hypertension, basic diagnostic tests that should be ordered include a chest x-ray, electrocardiogram, urinalysis, complete blood count, chemistry profile, lipid profile, and thyroid-stimulating hormone level. Renal arteriogram, plasma renin activity, 24-hour urinary sodium, and echocardiogram are more expensive tests. They are indicated when hypertension is severe or refractory to treatment or when underlying renal pathology is suspected.

Murmurs are graded according to their intensity (loudness). A murmur that is audible with the stethoscope off the chest is a A. grade III murmur. B. grade IV murmur. C. grade V murmur. D. grade VI murmur.

grade VI murmur. Answer D The intensity of a murmur is determined by the quantity and velocity of blood flow across the sound-producing area, its distance from the stethoscope, and the type of tissue between the murmur and the stethoscope. A murmur that is audible with the stethoscope off the chest is a grade VI murmur. A grade I murmur is barely heard; grade II is quietly heard and is approximately the same tone level as the S 1 -S 2 ; grade III is heard; grade IV is loud; grade V is very loud; and grade VI is the loudest. A thrill may accompany murmurs of grades IV, V, and VI.

The cardinal sign of right-sided heart failure in infants and children is A. hepatomegaly. B. edema of the lower extremities. C. tachypnea. D. cyanosis.

hepatomegaly. Answer A Hepatomegaly is the cardinal sign of right-sided heart failure in infants and children. Edema of the lower extremities is indicative of right ventricular heart failure in older children and adults. Tachypnea is the cardinal sign of left-sided congestive heart failure in children. Cyanosis is a clue to the presence of heart disease but is not specifically a sign of right-sided heart failure.

When a client is getting ready for cardiac catheterization, which question is essential to ask? A. " Are you allergic to shellfish? " B. " Have you ever had a catheterization before? " C. " Have you completed an advanced directive? " D. " What current medications are you on? "

" Are you allergic to shellfish? " Answer A While all these questions are important, if your client is allergic to shellfish, she or he may be allergic to the dye used for cardiac catheterization. Of note, recent studies suggest that a true allergy to iodine is a medical myth. Many people are labeled as having an iodine allergy if they have a reaction to iodine-containing foods or drugs, such as seafood and Betadine. They are then sometimes denied radiocontrast dyes or other products that contain iodine, but this precaution is not usually necessary. Iodine molecules are too small to trigger an antibody-mediated allergic reaction. A reaction to iodine-containing foods or products is usually either caused by an agent other than the iodine or is not a true allergy. Seafood allergies are usually due to proteins in the seafood. Povidone-iodine (Betadine, etc.) reactions are usually due to irritation or contact dermatitis. Potassium iodide (Iosat, ThyroSafe, ThyroShield) reactions are extremely rare, less than one in a million. And it is likely that iododerma, an acnelike rash due to the large dose of iodine, is not a true IgE-mediated immune reaction. Radiocontrast dye reactions are often caused by the high osmolarity, not by the iodine. Explain that the radiologist can use a low -osmolarity contrast dye to lower the risk. Help prevent confusion by avoiding the term iodine allergy. Instead, refer to the specific reaction, such as seafood allergy, povidone-iodine dermatitis, or contrast-media sensitivity, especially when asking about allergies and making notes in client records.

For clients with known coronary artery disease, it is recommended that the low-density lipoprotein cholesterol be A. 200 mg/dL or more. B. 100 mg/dL or less. C. 101 - 130 mg/dL. D. 131 - 200 mg/dL.

100 mg/dL or less. Answer B According to the National Cholesterol Education Program guidelines, for clients with known coronary artery disease (CAD), it is recommended that the low-density lipoprotein (LDL) cholesterol level be 100 mg/dL or less. Studies, including the Scandinavian Simvastatin Survival Study and the West of Scotland Coronary Prevention Group Study, reported that lowering LDL cholesterol levels resulted in a significant reduction of vascular events such as myocardial infarction or cerebrovascular accident in individuals with and without CAD. It has likewise been shown that with an LDL level of 100 or lower, further progression of the disease process is arrested, and with LDL of 70 or lower, existing atherosclerotic plaque may actually undergo a degree of revision.

New recommendations from the American Heart Association suggest that clients older than 80 years maintain a systolic blood pressure at what level when no contraindications exist? A. Less than 100 B. Less than 110 C. 120 - 130 D. 140 - 145

140 - 145 Answer D A systolic blood pressure (BP) of 140 - 145 is the target for ages 80 and up. This range is associated with a lower cardiovascular risk than higher levels — and possibly fewer side effects than lower levels. But do not be concerned if clients over 80 are doing well with a systolic BP less than 140 mm Hg and a diastolic BP over 65 mm Hg.

To reduce the progression of atherosclerotic lesions and occlusions in postcoronary artery bypass graft clients, it is recommended that the low-density lipoprotein cholesterol level be aggressively reduced to A. 70 mg/dL. B. 80 - 100 mg/dL. C. 101 - 120 mg/dL. D. 121 - 140 mg/dL

70 mg/dL. Answer A A study by the Post Coronary Artery Bypass Graft Trial investigators found that aggressively lowering the low-density lipoprotein (LDL) cholesterol level to 70 mg/dL in clients who had previously undergone bypass surgery was more effective in reducing the progression of atherosclerotic lesions and occlusions than any other moderate treatment. Maintaining the LDL at 70 may reverse some atherosclerotic lesions. When the LDL is less than 70, studies suggest a degree of plaque reversibility may result.

Mr. Michaels has a long-standing cardiac problem. His electrocardiogram rhythm strip is shown below. Which medication should he be taking to prevent a pulmonary or cerebral problem? A. An ACE inhibitor, such as enalapril (Vasotec) B. An antiarrhythmic agent, such as procainamid (Procan-SR) C. An anticoagulant, such as warfarin or dabigatran D. An anticonvulsant, such as phenytoin (Dilantin)

An anticoagulant, such as warfarin or dabigatran Answer C Mr. Michaels ' s electrocardiogram rhythm strip shows atrial fibrillation. To prevent continuous release of microemboli into the circulation, resulting in a pulmonary embolism or cerebrovascular accident, Mr. Michaels will probably be taking warfarin or dabigatran. Dabigatran (Pradaxa) may be ordered instead of warfarin for nonvalvular atrial fibrillation because Dabigatran does not need monitoring. Dabigatran also prevents more strokes than warfarin — about 5 fewer strokes per 1,000 clients per year — with a similar overall bleeding risk. But there are safety concerns. Keep these nuances in mind. Switching from warfarin to dabigatran. Switch clients with poor international normalized ratio (INR) control or those who cannot be well monitored. To switch, stop warfarin and wait until the INR is below 2, then start dabigatran. There are cases of bleeding when dabigatran is started too soon. Prescribe 150 mg bid or 75 mg bid if creatinine clearance is 15 - 30 mL/min. Do not routinely switch clients who are doing well on warfarin. Dabigatran causes more gastrointestinal bleeds and may prevent fewer MIs. Missed doses. Encourage careful adherence to the twice-daily regimen. Dabigatran starts to wear off about 15 hours after the dose is taken. If clients miss a dose, advise taking it as soon as possible unless the next dose is due in less than 6 hours. Tell clients not to double up on doses. chapter 11: CARDIOVASCULAR PROBLEMS 281 Bleeding. Dabigatran does not have any antidote, such as vitamin K. For serious bleeding, stop dabigatran and give fresh frozen plasma or whole blood if necessary. Surgery. Stop dabigatran at least 1 day before surgery or, for clients with renal insufficiency, 3 days before surgery. Surgeons may stop it sooner for surgeries with a high bleeding risk. Dyspepsia. Dabigatran capsules contain tartaric acid to improve absorption, so dyspepsia is common. Taking them with food, an H2 blocker, or a proton pump inhibitor might help. PPIs may decrease absorption, but probably not enough to reduce dabigatran's efficacy. If dyspepsia persists, use warfarin instead. In addition, digitalis (Digoxin) is usually ordered, sometimes in combination with either a beta blocker or a calcium channel blocker. The antiarrhythmic agent procainamide (Procan-SR) is used for ventricular arrhythmias. Anticonvulsants, such as phenytoin, are not indicated.

For the routine management of heart failure, which one of the following medication classes would not be recommended? A. ACE inhibitor B. Beta blocker C. Calcium channel blocker D. Diuretic

Calcium channel blocker Answer C Calcium channel blockers, with the exception of amlodipine and felodipine, should not be ordered for the routine management of heart failure. The clinician must consider treatments that will decrease the cardiac workload, reduce mortality, and control atrial fibrillation. To reduce mortality, ACEIs should be ordered, which are indicated for all patients with LV systolic dysfunction, unless there are specific contraindications, such as history of intolerance or adverse reactions to these agents, serum K > 5.5 mEq, or symptomatic hypotension. If the patient cannot tolerate ACEIs, ARBs can be used. Vasodilators and nitrates can also be used if ACEIs are not tolerated. Diuretics should be added to this therapy if symptoms persist or volume overload

Headache, flushing, tachycardia, and peripheral edema are adverse effects associated with which class of antihypertensive agents? A. Beta blockers B. Calcium channel blockers C. ACE inhibitors D. Diuretics

Calcium channel blockers Answer B Headache, flushing, tachycardia, and peripheral edema are adverse effects associated with calcium channel blockers. Calcium channel blockers may antagonize any of several cell membrane calcium entry channel receptors. As a result, some of these agents can express different effects on regional circulation. Although verapamil (Calan) and diltiazem (Cardizem) may reduce sinus rate, dihydropyridine calcium channel blockers may cause more potent peripheral vasodilation and can also cause headache, flushing, and tachycardia. Calcium channel blockers may also cause peripheral edema from reflex postcapillary constriction, which causes an increase in capillary hydrostatic pressure. This leads to movement of intravascular fluid to peripheral tissues and edema that is unassociated with weight gain.

Individuals with clinical evidence of chronic ischemic heart disease (abnormal electrocardiogram, chest pain syndrome, unusual dyspnea, or fatigue) should have a full evaluation done to evaluate their risk for myocardial infarction or sudden death. When initially collecting data for this risk stratification, which would be the last step to be considered if at all? A. Careful history and physical examination B. Exercise stress testing with or without nuclide imaging C. Cardiac catheterization D. Chest x-ray

Cardiac catheterization Answer C Cardiac catheterization is not generally performed during initial data collection. Selected individuals may require catheterization for severe or extremely unstable symptoms; however, angiography is generally reserved for those with positive exercise stress tests. The initial history and physical examination should explore severity of symptoms, functional disability, quality of life, and cardiac risk factors. This information assists the provider in identifying high-risk individuals who would benefit from further diagnostic testing. Electrocardiographic stress testing has a high false-positive rate. The use of thallium or sestamibi agents with exercise testing has a lower frequency of false-positive results. Nuclide imaging may be used for initial evaluation of clients or to confirm those with a questionable history and a positive exercise stress test. A chest x-ray can identify any structural abnormalities such as cardiomegaly or aortic aneurysm that suggest underlying cardiovascular pathology.

During a cardiovascular assessment, why is it important to note funduscopic changes? A. You may pick up beginning cataracts in this age group. B. You should note any glaucoma because some medications will be contraindicated. C. Changes may suggest the possibility of target organ involvement. D. You may be able to pick up arcus senilis.

Changes may suggest the possibility of target organ involvement. Answer C Because hypertension and diabetes are risk factors for coronary artery disease, it is important to note any funduscopic changes early in the progression of the disease.

Which of the following is the most important preventable cause of premature death in women? A. Hypertension B. Obesity C. Cigarette smoking D. Alcoholism

Cigarette smoking Answer C Cigarette smoking is the most important preventable cause of premature death in women. Cigarette smoking increases cerebrovascular disease (CVD) incidence and mortality. More than 50% of MIs in women younger than age 50 years are related to smoking. Women who smoke and use oral contraceptives increase their risk even more, while smoking cessation can lower their risk of CVD by 33% within 2 years.

Many older adults develop postural hypotension with hypertensive drug therapy. What is included in your teaching with these individuals? A. Drug therapy will be discontinued as soon as their blood pressure stabilizes to prevent this problem from recurring. B. Slight dehydration will prevent postural hypotension from occurring. C. Clients should sleep in a high Fowler ' s position to prevent this from happening. D. Clients should be taught to sit on the edge of the bed before standing.

Clients should be taught to sit on the edge of the bed before standing. Answer D Because some older adults develop postural hypotension with hypertensive drug therapy, the clinician must insist that these clients change position slowly while on HTN medication and that they sit on the edge of the bed for several minutes before standing. In addition, clients with postural hypotension should be urged to avoid volume depletion by drinking adequate quantities of water. Lifestyle modifications cannot be stressed enough.

Which of the following statements regarding the Joint National Committee (JNC) on Prevention, Detection, Evaluation, and Treatment of High Blood Pressure category about prehypertension is true? A. Clients with prehypertension usually remain in that category forever. B. Clients with a blood pressure (BP) in the range of 130/80 to 139/89 mm Hg are twice as likely to develop hypertension as those with lower values. C. All clients in this category should be started on diuretics immediately to avoid future end-organ disease. D. Diastolic BP control should be the focus of treatment.

Clients with a blood pressure (BP) in the range of 130/80 to 139/89 mm Hg are twice as likely to develop hypertension as those with lower values. Answer B The JNC includes a category designated as prehypertension. The range for the systolic pressure is 120 - 139 and for diastolic pressure is 80 - 89. Clients with prehypertension are at an increased risk for progression to hypertension. Clients in the 130/80 to 139/89 mm Hg blood pressure (BP) range have twice the risk of developing hypertension as those with lower values. Diagnostic work-up of HTN should includes the following: Assess risk factors and comorbidities Reveal identifiable causes of HTN Assess presence of target organ damage (TOD) Conduct history and physical exam Obtain lab studies to include U/A, blood glucose, Hct, lipid panel, serum potassium, creatinine and calcium Electrocardiogram and echo as presentation dictates NOTE: Latest studies from AHA and ACC: Hypertension is very common among older adults. Sixty-four percent of older men and 78% of older women have high BP, placing them at heightened risk for heart disease, including heart failure, stroke, coronary artery disease, and atrial fibrillation, as well as chronic kidney disease and diabetes mellitus. Despite its prevalence, rates of BP control remain substantially lower in the elderly than in younger clients. In fact, over age 80, only one in three men and one in four women have adequate control of their BP. Faced with an aging client population and compelling data that confirm the benefits of BP-lowering medications in the elderly (80 or more years), the American College of Cardiology (ACC) and the American Heart Association (AHA) released the first expert consensus document to help clinicians reduce the risks for developing and effectively manage hypertension in older adults. There has been uncertainty about the appropriate therapeutic target for clients 80 years and older. Levels of less than 140/90 mm Hg in persons 65 - 79 years and a systolic BP between 140 and 145 mm Hg in persons 80 years and older if tolerated were discussed; hypertension in older adults is usually characterized by an elevated systolic BP and a normal or low diastolic BP due to ageassociated stiffening of the large arteries. Medications should be used as appropriate. Angiotensin converting enzyme (ACE) inhibitors, beta blockers, angiotensin receptor blockers, diuretics, and calcium channel blockers are all effective in lowering BP and reducing cardiovascular outcomes among the elderly; clinicians should select medications based on efficacy, tolerability, specific comorbidities, and cost. For example, a person who has had a heart attack should be started on a beta blocker and an ACE inhibitor.

When performing a cardiac assessment, where is the most essential site for assessing edema? A. Dependent areas B. Periorbital areas C. Upper extremities D. Cerebral edema

Dependent areas Answer A When assessing for edema, check the client ' s most dependent areas such as the legs, sacrum, and scrotum. Peripheral edema is the accumulation of fluid within the interstitial spaces of the extremities. When the edema involves the lower extremities, it is a symptom of an underlying disorder; it may be caused by cardiac conditions (e.g., heart failure, chronic venous insufficiency, or thrombophlebitis), renal and hepatic disease, or by trauma, tumors, or inflammation. Anasarca is generalized body edema, which may indicate right-sided CHF.

Before counseling partners about sexual activity following a myocardial infarction, the provider should consider what factor(s)? A. Most clients do not want to know how their condition affects their sex life. B. Spouses are knowledgeable about their partner's condition; therefore, they do not need counseling. C. Most clients return to the same frequency of sexual intercourse after they have regained their physical strength. D. Depression, loss of interest, spousal reluctance, and anxiety may interfere with a client ' s resumption of sexual activities.

Depression, loss of interest, spousal reluctance, and anxiety may interfere with a client ' s resumption of sexual activities. Answer D Before counseling partners about sexual activity following a myocardial infarction (MI), the provider should consider that depression, loss of interest, spousal reluctance, and anxiety may interfere with a client ' s resumption of sexual activities. The diagnosis of heart disease affects the spouse as well as the client. Spouses may display anxiety and depression yet often are not included in the assessment, counseling, and treatment processes. The level of sexual activity reported 1 year after an MI is only about 60% of the level before illness.

Which classification of antihypertensive drugs is the most effective for treating hypertension in African American clients and older adults? A. Diuretics B. ACE inhibitors C. Beta blockers D. Alpha-adrenergic blockers

Diuretics Answer A Diuretics are effective in lowering blood pressure in all clients, especially African Americans and older adults. Numerous studies have shown reductions in morbidity and mortality with these agents. Relative to beta blockers and ACE inhibitors, diuretics are more potent in African Americans, older adults, obese clients, and other subgroups who exhibit increased plasma volume or low plasma renin activity. Alpha-adrenergic blockers may be used effectively if the client has coexisting prostatism.

Ted, age 18, is to have a cardiac screening examination to determine if he can play college basketball. The diagnostic test of choice for detecting hypertrophic cardiomyopathy or idiopathic left ventricular hypertrophy is a(n): A. echocardiogram. B. electrocardiogram. C. arteriogram. D. stress test.

Echocardiogram Answer A In athletes younger than age 30, the three most common causes of sudden death are hypertrophic cardiomyopathy, idiopathic left ventricular hypertrophy, and coronary artery anomalies. The diagnostic test that should be employed to detect a potential problem is an echocardiogram, which many colleges are now instituting as part of their screening criteria for athletes. The echocardiogram will detect hypertrophic cardiomyopathy and idiopathic left ventricular hypertrophy. The arteriogram is a reliable test available to detect coronary artery anomalies. An alternative to arteriogram for the diagnosis of CAD is the CT angiogram coronary computed tomography angiogram. • A coronary computed tomography angiogram (CTA) uses advanced CT technology, along with intravenous (IV) contrast material (dye), to obtain high-resolution, three-dimensional pictures of the moving heart and great vessels. • CTA is also called multislice computed tomography (MSCT), cardiac CT, or cardiac CAT. During CTA, x-rays pass through the body and are picked up by detectors in the scanner, three-dimensional images on a computer screen. These images enable physicians to determine whether plaque or calcium deposits are present in the artery walls. • CTA is used as a noninvasive method for detecting blockages in the coronary arteries. A CTA can be performed much faster (in less than 1 minute) than a cardiac catheterization, with potentially less risk and discomfort as well as decreased recovery time. An electrocardiogram will diagnose rhythm disturbances. A stress test will record the effects of stress on the heart.

Which of the following drugs should be considered as first-line therapy for a client with hypertension and heart failure? A. Enalapril (Vasotec) B. Diltiazem (Cardizem) C. Atenolol (Tenormin) D. Metoprolol (Lopressor)

Enalapril (Vasotec) Answer A ACE inhibitors such as enalapril (Vasotec) should be considered as first-line therapy for a client with hypertension (HTN) and heart failure. Research from the Framingham Heart Study demonstrated that HTN continues to be the major cause of left ventricular failure in the United States. In treating heart failure, ACE inhibitors alone or with a diuretic are effective in reducing morbidity and mortality. Evidence suggest beta blockers, such as carvedilol and metoprolol succinate, are of great value in managing stable systolic dysfunction heart failure and decreasing morbidity and mortality. Alternative medications, such as a nitrate and hydralazine combination, are effective for clients who develop hyperkalemia or renal dysfunction with ACE inhibitors or angiotensin receptor blockers. Contingent on the etiology of the heart failure (such as systolic versus diastolic dysfunction), additional medications may include metoprolol succinate (Toprol XL) or carvedilol (Coreg) in systolic dysfunction or isosorbide dinitrate (Isordil) and hydralazine (Apresoline) for diastolic dysfunction.

When auscultating a client's heart, you note a short, high-frequency click (opening snap) after S2 during the beginning of diastole. What could this indicate? A. Aortic regurgitation B. Mitral stenosis C. Mitral regurgitation D. Nothing; this is normal

Mirtral stenosis Answer B When auscultating the heart, if you note a short, high-frequency click (opening snap) after S 2 during the beginning of diastole, suspect mitral stenosis. Mitral stenosis occurs with a stiff stenotic valve. The valve opening becomes restricted, impeding forward flow. The stenosis generates increased left atrial pressures, which are necessary to overcome the increased resistance to flow. The " snap " is created when the mitral valve leaflet is rapidly reversed toward the left ventricle in early diastole (as left ventricular pressures are reduced) by the high left atrial pressures.

Sheila, age 78, presents with a chief complaint of waking up during the night coughing. You examine her and find an S 3 heart sound, pulmonary crackles (rales) that do not clear with coughing, and peripheral edema. What do you suspect? A. Asthma B. Nocturnal allergies C. Heart failure D. Valvular disease

Heart failure Answer C The greater the number of symptoms in a client, the more reliable is the diagnosis of heart failure. One of the classic symptoms of heart failure that Sheila has is a nocturnal cough that wakes her up. Other findings indicative of heart failure include an elevated jugular venous pressure, an S 3 heart sound, a laterally displaced apical impulse, pulmonary crackles (rales) that do not clear with coughing, and peripheral edema that is not caused by venous insufficiency. A persistent night cough is often the only symptom seen with some types of asthma, but the cough is dry without any of the other symptoms that Sheila has. Allergies may present as a dry cough at night with a postnasal drip. Valvular heart disease may present with dyspnea, orthopnea, and paroxysmal nocturnal dyspnea but not a cough with crackles (rales).

Which resuscitation recommendation made by the American Heart Association has the highest priority? A. Timely delivery of an epinephrine bolus B. Rapid provision of advanced airway management C. High quality, uninterrupted chest compression D. Early electrical therapy

High quality, uninterrupted chest compression Answer C The American Heart Association and an International Liaison Committee determined in the 2011 guidelines of evidence-based practice that the primary goal in a resuscitation event is the provision of high-quality chest compression. The only other intervention noted to increase survival and neurologic recovery up to 1 year after resuscitation was the timely appropriate provision of hypothermia after return of spontaneous circulation (ROSC).

Which statement regarding rheumatic fever in children is true? A. The peak period of risk is ages 1 - 5 years. B. The disease is more common in boys. C. The disease is more common in whites. D. Group A beta-hemolytic streptococcal infections of the upper respiratory track remain the primary environmental trigger of rheumatic fever in children.

Group A beta-hemolytic streptococcal infections of the upper respiratory track remain the primary environmental trigger of rheumatic fever in children. Answer D Group A beta-hemolytic streptococcal infections of the upper respiratory tract remain the primary 288 unit three: ASSESSMENT AND MANAGEMENT OF CLIENT ILLNESS environmental trigger that acts on predisposed children to cause rheumatic fever and subsequent cardiovascular disease. Although the incidence of the disease has significantly declined over the past half century, there has been a resurgence in several regions of the United States since the mid-1980s. Children between the ages of 5 and 15 are at most risk; blacks are slightly more at risk than whites, and girls are more at risk than boys. In general, once the condition has been diagnosed, antibiotic therapy is indicated. Penicillins Penicillin remains the treatment of choice for group A streptococcal upper respiratory tract infections, since it is the only antibiotic that has been evaluated in controlled studies. A single injection of intramuscular benzathine benzylpenicillin is the most effective treatment in eradicating group A streptococci, probably because of its long duration of action. It can also be used for mass prophylaxis. Oral phenoxymethylpenicillin administration for streptococcal pharyngitis must be continued for 10 days. Other orally administered penicillins include ampicillin, amoxicillin, and the semisynthetic penicillins. Macrolides For penicillin-allergic clients, treatment with oral erythromycin for 10 days is often used. Newer macrolides are reported to be associated with fewer adverse effects but are generally more expensive. Short-course therapy with these newer macrolides is effective, but more definitive data are required before use in primary prevention is recommended. Cephalosporins First- and second-generation cephalosporins have been used to treat group A streptococcal infections. As a rule, cephalosporins are more expensive than penicillin. Short-course therapy (less than 10 days) with some cephalosporins is also under evaluation.

Your client has documented hypertension (blood pressure of 140/92 mm Hg confirmed on multiple visits) and no other medical history. What would you consider low down on the list of priorities? A. Immediate initiation of antihypertensive drug therapy to prevent any complications B. Identification of known causes of hypertension C. Assessment of the presence or absence of target organ damage and the extent of the disease D. Identification of clinical cardiovascular disease (CVD) and risk factors, as well as other concomitant disorders that may guide prognosis and treatment

Immediate initiation of antihypertensive drug therapy to prevent any complications Answer A Treatment of hypertension is dependent on risk stratification. Full evaluation of the client is necessary to determine the presence of target organ damage (TOD), cardiovascular disease (CVD), and major risk factors. Also, those with identifiable causes should have the underlying problem addressed. If the client has no risk factors (except diabetes) and no TOD or CVD, the client would initially be started on lifestyle modification. The Joint National Committee on Prevention, Detection, Evaluation, and Treatment of High Blood Pressure recommends that individuals who have high-normal blood pressure, as well as known renal insufficiency, heart failure, or diabetes mellitus, should be considered for prompt pharmacological therapy and lifestyle modifications

Jim, age 72, has a history of type 2 diabetes mellitus that has been controlled by diet. He has come for a routine examination and reports feeling more tired than usual. On his electrocardiogram (ECG), you notice Q waves in leads II and III, and he is in atrial fibrillation that was not present on his previous ECGs. What do you do? A. Immediately hospitalize Jim and order a cardiology consultation, start intravenous administration of an anticoagulant and nitrates, and run serial cardiac enzyme tests. B. You do nothing because you know the normal progression of type 2 diabetes in older adults includes changes in their ECG as a result of neuropathy involving the transmission of electrical impulses. C. Initiate aspirin therapy and refer Jim to a cardiologist for evaluation of occult ischemic heart disease and left ventricular function as soon as possible. Initiate lipid-lowering therapy because Jim may have had an acute myocardial infarction. D. Initiate lipid-lowering therapy because Jim may have had an acute myocardial infarction.

Initiate aspirin therapy and refer Jim to a cardiologist for evaluation of occult ischemic heart disease and left ventricular function as soon as possible. Initiate lipid-lowering therapy because Jim may have had an acute myocardial infarction. Answer C Older adults and clients with diabetes frequently demonstrate defective anginal warning systems — a result of altered cardiac neural pathways (older age, diabetics), physical interruption of nerve pathways (cardiac transplant or bypass surgery), an abnormally high pain perception threshold, or milder degrees of ischemia. Because Jim is an older adult with diabetes, he may have had a silent myocardial infarction (MI), as well as silent ischemia. Neither his electrocardiogram nor his clinical presentation suggests an acute MI, so hospitalization is not indicated. However, aspirin therapy (unless contraindicated for other reasons) is indicated because it has been shown to be an independent factor in reducing the risk of subsequent MI. Work-up for a possible silent MI should include, when clinical status is stable, exercise stress testing with thallium imaging to determine ischemic burden and changes indicative of MI. Evaluation for left ventricular wall motion abnormalities and ejection fraction with echocardiography may also confirm previous MI or the presence of ischemia. Likewise, the client has a newly developed atrial fibrillation, which has the propensity to alter cardiac output and forward flow with resultant decreases in peripheral perfusion as well as possible subsequent heart failure. An electrophysiology (EP) consult may provide benefit, as a return to normal sinus rhythm and possible cardiac resynchronization therapy (CRT) or cardiac resynchronization therapy defibrillator (CRT-D) may optimize clinical status contingent on the results of the echo and QT interval.

Sarah, who is postmenopausal, has controlled asthma, hypertension being effectively treated with medication, and smokes cigarettes. She has a low-density lipoprotein (LDL) cholesterol level of 170 mg/dL and a high-density lipoprotein (HDL) cholesterol level of 40 mg/dL. To reduce Sarah ' s risk of a coronary event, the treatment plan would focus on A. lowering her LDL cholesterol level. B. lowering her HDL cholesterol level. C. aggressively treating and controlling her hypertension and asthma. D. getting Sarah to stop smoking.

Lowering her LDL cholesterol level Answer A According to the National Cholesterol Education Program (NCEP) expert panel, Sarah ' s treatment should focus on lowering her low-density lipoprotein (LDL) cholesterol to less than 100 mg/dL because she has several risk factors for coronary artery disease. Certainly treating her hypertension and asthma and assisting her with a smoking cessation treatment program will also help. Further recommendations may include weight management, encouraging physical activity on most days of the week. The powerhouse medications for the reduction of LDL cholesterol remains the hydroxymethylglutaryl-coenzyme A reductase inhibitors (statins). Note that there is a slight risk of rise in hepatic enzymes with statin usage. Baseline levels should be obtained and levels should be reassessed as per product recommendation.

Selma has acute peripheral arterial occlusion of a lower extremity. Before you begin your examination, you know that it: A. may present with only complaints of coldness or paresthesia of the extremity. B. may present as the only disease. C. always occurs in the lower extremities. D. will result in an extremity that appears " blue. "

May present with only complaints of coldness or paresthesia of the extremity. Answer A The manifestation of acute peripheral arterial occlusion is classically a dramatic one with sudden pain, pallor, paresthesia, paralysis, and pulselessness. However, it may also be more gradual and less dramatic, presenting with only complaints of coldness or paresthesia of the extremity. The occlusion may be caused by previously unknown cardiac disease, a vascular lesion, or an occult systemic disease, such as connective tissue diseases or arteritis, and should serve as a clue to look for other conditions because it rarely occurs by itself. Occlusions can occur in upper or lower extremities.

Jessica is pregnant and is being seen for the first time. She states that she was told that her blood pressure (BP) has been high for a long time, but she never wanted to take medication for it. Today her BP is 172/98, her lungs are clear, and she has no pedal or ankle edema. What medication would you most likely order? A. Methyldopa (Aldomet) B. Atenolol (Tenormin) C. Nifedipine (Adalat, Procardia) D. Clonidine (Catapres)

Methyldopa (Aldomet) Answer A Methyldopa, an alpha-adrenergic inhibitor, is used to treat chronic hypertension during pregnancy. At low doses, it can be used as monotherapy. At higher doses, it may cause sodium and fluid to accumulate, so diuretic therapy may be required. Blood pressure (BP) drops during normal pregnancy, and no data suggest the use of medication in clients with BP less than 160/100 mm Hg. Jessica, however, has a history of high BP, and today it is high as well. Methyldopa has a pregnancy rating of B, meaning that studies in humans have found no harm to the fetus, but studies in other animals have shown harm to the fetus, or there are no reliable studies in humans. Atenolol (Tenormin) has been associated with mild intrauterine growth restriction when used in randomized trials focusing on the treatment of chronic hypertension during pregnancy. It has a category rating of D (unsafe in pregnancy). Nifedipine (Adalat, Procardia) is used occasionally to treat preterm contractions and is rated as category C (safety for use during pregnancy has not been established). Clonidine (Catapres) is usually a third-line agent if other medications cannot be tolerated. It is rated as category C in pregnancy

Mort is hypertensive. Which of the following factors influenced your choice of using an alpha blocker as the antihypertensive medication? A. Mort is black. B. Mort also has congestive heart failure. C. Mort has benign prostatic hyperplasia (BPH). D. Mort has frequent migraine headaches.

Mort has benign prostatic hyperplasia (BPH). Answer C An alpha blocker is the antihypertensive agent of choice because Mort has benign prostatic hyperplasia (BPH). In addition to lowering the client ' s blood pressure, an alpha blocker will provide symptomatic relief of his BPH. Calcium channel blockers or diuretics are effective antihypertensive agents in blacks, whereas a beta blocker would be the drug of choice if Mort had migraine headaches in addition to his BPH. Diuretics and ACE inhibitors are the drugs of choice for treating congestive heart failure with accompanying hypertension.

Which is the most common symptom of digitalis toxicity? A. Nausea and vomiting B. Tingling of the extremities C. Rash D. Headache

Nausea and vomiting Answer A The most common symptom of digitalis toxicity is nausea and vomiting. Other adverse side effects may include, headache, diarrhea, stomach pain, loss of appetite, unusual tiredness or weakness, slow heartbeat, palpitations, irregular heartbeat, drowsiness, confusion, fainting, changes in vision including seeing a halo or light around objects, and disorientation. The symptoms usually occur before any cardiotoxic effects take place and are warning signs that the dosage needs to be adjusted. Tingling of the extremities is not a symptom of digitalis toxicity.

Nicotinic acid is an inexpensive drug used to treat serum hyperlipidemia. Which of the following statements is true about nicotinic acid? A. Nicotinic acid lowers low-density lipoprotein cholesterol levels, raises high-density lipoprotein cholesterol levels, and decreases triglyceride levels. B. Nicotinic acid is the drug of choice for individuals with diabetes. C. Nicotinic acid may decrease the effect of some antihypertensive agents. D. Rare adverse reactions of nicotinic acid include flushing, pruritus, and gastrointestinal upset.

Nicotinic acid lowers low-density lipoprotein cholesterol levels, raises high-density lipoprotein cholesterol levels, and decreases triglyceride levels Answer A Nicotinic acid lowers low-density lipoprotein cholesterol levels, and is effective in the management of the highly atherogenic lipoprotein. It raises high-density lipoprotein cholesterol levels and decreases triglyceride levels; may potentiate the effect of some antihypertensive agents; and may commonly (not rarely) cause adverse reactions including flushing, pruritus, and gastrointestinal upset. Nicotinic acid is not the drug of choice for individuals with diabetes. A potential adverse effect of nicotinic acid is impairment of glucose intolerance; therefore, it should be used with caution in clients with diabetes. If nicotinic acid is administered, serum glucose and liver function tests should be closely monitored. Nicotinic acid should likewise be avoided in clients with gout and/or peptic ulcer disease.

Greg has just been given a diagnosis of congestive heart failure. Which of his medications should be discontinued? A. Nifedipine (Procardia XL) for long-term management of his chronic stable angina B. Hydrochlorothiazide (HydroDIURIL) for his hypertension C. Enalapril (Vasotec) for his hypertension D. Butalbital (Esgic) for his headaches

Nifedipine (Procardia XL) for long-term management of his chronic stable angina Answer A Nifedipine (Procardia XL), a calcium channel blocker, should be discontinued, along with most antiarrhythmic agents, when a client develops congestive heart failure (CHF) because both of these classes of medications are important causes of worsening heart failure. However, amiodarone is an antiarrhythmic used successfully in the treatment of clients with CHF. Diuretics are the most effective means of providing symptomatic relief in clients with CHF, so Greg should stay on his hydrochlorothiazide (HydroDIURIL). ACE inhibitors, such as enalapril (Vasotec), should be the initial treatment, along with diuretics in most symptomatic clients, so Greg can also continue taking this. The barbiturate butalbital (Esgic) is not contraindicated in clients with CHF.

Which of the following conditions would warrant bacterial endocarditis prophylaxis? A. Prosthetic heart valves B. Surgical repair of atrial septal defect C. MVP without significant mitral regurgitation D. Cardiac pacemakers

Prosthetic heart valves Answer A Bacterial endocarditis prophylaxis is recommended for high-risk clients, including those with prosthetic heart valves, previous bacterial endocarditis, complex cyanotic congenital heart disease, and surgically constructed systemic pulmonary shunts or conduits. Moderate-risk clients requiring this include those with most other congenital cardiac deformities, such as ventricular septal defect (VSD), rheumatic heart disease, and other acquired cardiac defects, like those successfully surgically repaired, hypertrophic cardiomyopathy, and mitral valve prolapse (with valvular regurgitation and/or thickened valves).

Martha, age 36, presents with a complaint of increasing shortness of breath and fatigue over the past 6 months. She has been trying to lose weight, has been on a walking exercise program for over a year, and had taken the fenfluraminephentermine (Fen-Phen) combination years ago but stopped when its adverse effects were reported. Your examination reveals a grade II/VI systolic murmur along the apex. What do you do? A. Obtain pulmonary function tests. B. Instruct the client about other exercise activities that may not produce her symptoms. C. Refer the client to a cardiologist for an echocardiogram and cardiovascular work-up. D. Start endocarditis prophylaxis.

Refer the client to a cardiologist for an echocardiogram and cardiovascular work-up. Answer C Martha should be referred to a cardiologist for an echocardiogram and cardiovascular work-up because she has taken the fenfluramine-phentermine (fen-phen) combination. The U.S. Department of Health and Human Services issued the following recommendations for individuals who took fenfluramine and dexfenfluramine: Persons who have had the drugs should undergo a careful history and cardiovascular examination by their health-care provider; and individuals with signs or symptoms suggesting valvular disease should have an echocardiogram.

The classic 12-lead electrocardiogram change(s) that indicate(s) an acute coronary syndrome is (are) A. ST-segment elevation. B. T-wave inversion. C. flipped P waves with a prolonged PR interval. D. deep Q waves.

ST-segment elevation. Answer A The classic 12-lead electrocardiogram (ECG) change that indicates an acute coronary syndrome (ACS) is ST-segment elevation. The American Heart Association, in its therapeutic recommendations, discusses ST-segment elevation myocardial infarction (STEMI) as well as non-ST-segment elevation myocardial infarction (NSTEMI) in defining optimal therapeutic management of an ACS client. It is important to remember an actively evolving infarction could be present with no electrocardiographic changes appreciated. When assessing a client with a suspected ACS, timely assessment and management is essential. Timely cardiac catheterization and laboratory assessments and possible percutaneous coronary intervention is essential in decreasing mortality and morbidity. Further therapeutics may likewise include but not be limited to aspirin, Plavix, beta blocker and ACE inhibitor, statin therapy, and smoking cessation therapy if indicated.

When teaching a client with hypertension about restricting dietary sodium, you would include which of the following instructions? A. Sodium restriction can cause serious adverse effects. B. Diets with markedly reduced intake of sodium may be associated with other beneficial effects beyond blood pressure control. C. Seventy-five percent of sodium intake is derived from processed food. D. A goal of 3 g of sodium chloride or 1.2 g of sodium per day is easily achievable.

Seventy-five percent of sodium intake is derived from processed food. Answer C When teaching a client with hypertension about restricting dietary sodium, it is important to stress that 75% of sodium intake is derived from processed food. Although concern about severe sodium restriction has been raised, there is no evidence that lower levels of sodium intake cause any hazards. A sodium reduction to approximately 6 g of sodium chloride or 2.4 g of sodium a day is recommended for most clients and is easily achievable. The bottom line to stress to the client is that reducing the sodium will reduce water retention, resulting in less burden on the heart.

Characteristics of ischemic arterial ulcers include: A. an irregularly shaped border with crusting or scaling at the edges. B. severe pain. C. a location anywhere on the leg. D. a moist ulcer base with ill-defined borders.

Severe pain Answer B Characteristics of ischemic arterial ulcers include severe pain and a discrete border (which may have a " punched-out " appearance) with a pale, dry ulcer base and a slightly inflamed " halo " around the border if infected. They are typically located distally and are usually a result of trauma.

A blood pressure of 160/100 mm Hg is classified as A. prehypertension. B. stage 1 hypertension. C. stage 2 hypertension. D. stage 3 hypertension.

Stage 2 HTN Answer C In the JNC guidelines, hypertension stages 2 and 3 have been combined. A blood pressure of 160 or greater systolic and 100 or greater diastolic is considered to be stage 2 hypertension. Pharmacological therapy is definitely indicated. Several drugs are usually required for management of this stage. Hypertensive clients should have a goal BP of under 140/90 or, if they have diabetes or chronic kidney disease, under 130/80. Choice of appropriate antihypertensives should include a review of client comorbidities. With cardiac disease, the client would most likely benefit from a beta blocker. If the cardiac disease is associated with heart failure, the choice of beta blocker should be narrowed to Coreg or Toprol XL. With heart failure and diastolic dysfunction, exquisite hemodynamics must be achieved to optimize cardiac function, so the addition of hydralazine as well as isosorbide dinitrate (Isordil) and spironolactone (Aldactone) may be considered. If the client has diabetes mellitus or low-grade renal insufficiency and the serum creatinine remains controlled, the choice may be an ACE inhibitor. Calcium channel blockers of the dihydropuridines subcategory (Norvasc) may also be considered.

Rick is modifying his diet to try to lose weight, but after 3 months, he has not lost any weight, even though he has complied with his diet plan. A follow-up lipid profile reveals the following: total cholesterol, 238 mg/dL; triglycerides, 100 mg/dL; high-density lipoprotein cholesterol, 28 mg/dL; and low-density lipoprotein cholesterol, 190 mg/dL. What would you recommend? A. Continuing the diet plan for another month B. Starting an exercise program with a goal of uninterrupted aerobic exercise for 30 minutes 2 days a week C. Stopping his current diet plan and trying another D. Starting hypolipidemic drug therapy

Starting hypolipidemic drug therapy Answer D Because dietary therapy has not resulted in any significant reduction of Rick ' s low-density lipoprotein (LDL) cholesterol level, starting him on hypolipidemic drug therapy is strongly recommended to help him achieve an LDL cholesterol goal of less than 130 mg/dL. To further help reduce the risk of heart disease, regular physical activity of moderate intensity (the minimal goal is 30 minutes, five times a week), along with lowering blood pressure and reducing weight, is important. No further diagnostic intervention is indicated unless the client begins to complain of symptoms of ischemia, such as chest pain, increasing fatigue, or dyspnea.

A newly discharged outpatient surgery client presents with insidious onset of edema and dusky blue discoloration of the head and upper extremities. You know it is a medical emergency and suspect which of the following? A. Evolving cerebral infarction B. Impending myocardial infarction C. Superior vena cava syndrome D. Temporal arteritis

Superior Vena Cava Syndrome Answer C Superior vena cava syndrome occurs when there is obstructed venous return from the upper extremities, head, and neck region. Symptoms include edema and dusky blue discoloration of the upper extremities and head. Both the location of the obstruction and the rapidity with which it develops will determine the severity of symptoms. Diagnostic studies include chest radiography, CT, and venography. A CT scan is the most widely used. The most common cause of superior vena cava syndrome is cancer. Primary or metastatic cancer in the upper lobe of the right lung can compress the superior vena cava. Lymphoma or other tumors located in the mediastinum can also cause compression of the superior vena cava. Less often, the superior vena cava can become blocked with a blood clot from within. As more invasive medical procedures are being performed on clients, this cause of superior vena cava syndrome is being seen more frequently. Blood clot (thrombus) formation that causes superior vena cava syndrome is a complication of pacemaker wires, dialysis, and other intravenous catheters that are threaded into the superior vena cava (SVC). Signs and symptoms of SVC Syndrome may include shortness of breath and swelling of the arms and face. The symptoms occur because blood cannot return to the heart. Management of SVC is directed at managing the underlying cause and consists of various measures aimed at decreasing the severity of the obstruction.

Which statement about mitral valve prolapse (MVP) is true? A. MVP occurs in about 10% of the population. B. MVP is usually detected in older adults. C. The incidence is equal in men and women younger than age 20. D. The incidence is more common in women younger than age 20.

The incidence is more common in women younger than age 20. Mitral valve prolapse is more common in women younger than age 20 than in other populations. MVP occurs in about 2% - 4% of the population and is usually detected in young adulthood. The incidence is equal in men and women after age 20. Physical assessment findings demonstrate a murmur followed by a midsystolic click. When evaluating the client with a diagnosis of MVP, a review of systems should review any symptoms of heart disease. Clients should be questioned about any development of chest pain, palpitations, or syncope and asked if they have developed any activity intolerance or any recent weight gain, ankle edema, abdominal fullness or distention, or any dyspnea on exertion (DOE).

Which statement is true of hydroxymethylglutaryl-coenzyme A reductase inhibitors (statins)? A. They are the first drugs of choice for men younger than age 45. B. They should be given in the morning after breakfast. C. They may cause myopathies, especially at higher dosages or in combination with certain drugs. D. They are contraindicated in clients taking Coumadin.

They may cause myopathies, especially at higher dosages or in combination with certain drugs. Answer C Hydroxymethylglutaryl-coenzyme A reductase inhibitors (statins) may cause — albeit rarely — myopathies, especially at higher dosages or in combination with certain drugs, such as gemfibrozil (Lopid), nicotinic acid (Nicobid), or erythromycin (E-Mycin). Any reports of muscle pain and weakness should be evaluated. According to National Cholesterol Education Program guidelines, bile acid sequestrants are preferred over statins for primary prevention of coronary events in men younger than age 45 and women younger than age 55. The liver is more active in the production of cholesterol during the evening hours; therefore, the recommendations for all statins (except atorvastatin [Lipitor], which has a longer half-life) is for dosing to be done in the evening or at bedtime. Statins may increase the international normalized ratio and require an adjustment of Coumadin dosage; however, Coumadin use is not a contraindication to use of statins.

Which of the following conditions is the least frequent cause of heart failure? A. Hypertension B. Aortic stenosis C. Ischemic cardiomyopathy D. Valvular heart disease (mitral and tricuspid)

Valvular heart disease (mitral and tricuspid) Answer D Valvular heart disease (mitral and tricuspid) has become the least common cause of heart failure of the conditions listed because of the declining incidence and severity of rheumatic fever. On the other hand, aortic stenosis occurs frequently and is reversible. The most common cause of heart failure is ischemic cardiomyopathy. Systemic hypertension remains a common cause of congestive heart failure.

While much teaching is needed for your client with congestive heart failure (CHF), the most beneficial thing you can tell him that might prevent rehospitalization may be A. " Be sure to use two pillows at night. " B. " Weigh yourself every day and if your weight increases by 3 lb in 24 hrs or 5 lb in one week, call me. " C. " Take your pulse daily; if it increases by six beats, call me. " D. " Let me know if you ' re sleeping more than 10 hours a day and feel depressed. "

Weigh yourself every day and if your weight increases by 3 lb in 24 hrs or 5 lb in one week, call me. " Answer B Despite the technological advances of recent years, including cardiac resynchronization, implantable defibrillators, left ventricular assist devices, and totally implantable artificial hearts, it should be remembered that many clients with chronic heart failure are elderly and have multiple comorbidities. Clients who gain 3 lb in 1 day or 5 lb in 1 week should be seen immediately, which might prevent rehospitalization. Most clients require pillows at night and will probably have a recumbent cough, but the fluid overload characteristic of congestive heart failure (CHF) will be manifested by the weight gain. These clients may be depressed, thus sleeping more, and although treating the depression is important, it will not prevent a rehospitalization due to worsening CHF. Additional symptoms to advise clients to monitor for may include lower extremity edema, abdominal distention, jugular venous distention, palpitations, and increasing dyspnea on exertion. As clients are managed on an outpatient basis, they should be fully educated on interventions that will assist in improving presentation. Some of the educational discussions may include maintaining a low-sodium diet, avoidance of alcohol, daily weights, and daily blood pressure monitoring. Frequencies of visits to a heart failure specialist will be determined on an individual basis because titration of medications requires close assessment to optimize outcomes.

Which of the following usually indicates hyperlipidemia? A. Lipoma B. Xanthelasma C. Jaundiced skin D. Multiple actinic keratoses

Xanthelasma Answer B Xanthelasmas (small, yellow, raised plaques on the eyelids) are lipid deposits that may indicate hyperlipidemia.

Charles has chronic ischemic heart disease and is taking a beta blocker, which results in A. an increase in high-density lipoprotein cholesterol. B. a reduced heart rate. C. a decreased diastolic filling time. D. An increase in oxygen demand.

a reduced heart rate Answer B Beta blockers reduce the heart rate, which results in a decrease in oxygen demand and an increase in coronary blood flow from the increased diastolic filling time. Studies have also shown that beta blockers reduce mortality in clients who have had acute myocardial infarctions and reinfarction. Adverse effects of all beta blockers include a decrease in high-density lipoprotein cholesterol, bronchospasm, fatigue, sleep disturbance, worsening of congestive heart failure, gastrointestinal disturbances, and impotence.

Martin, age 56, has hypertension and has been taking antihypertensive medication for about 10 years. He has been very stable. You have not seen him in about 6 months. His examination today should specifically A. include only a blood pressure measurement with the client seated comfortably. B. include a funduscopic examination. C. be a focused examination limited to the respiratory and cardiovascular systems. D. include a discussion of weaning him off his medication.

include a funduscopic examination. Answer B Funduscopic examination for hypertensive retinopathy (arteriovenous nicking, arteriole narrowing, hemorrhages, exudates, and disk edema) should be included in the physical examination of a client with hypertension (HTN). When examining the client with HTN, at least two blood pressure measurements separated by 5 minutes should be performed with the client seated. An additional blood pressure reading taken with the client standing should also be considered for evaluation of " white coat HTN. " Further diagnostic work-up should include assessment of risk factors and comorbidities. Assess for identifiable causes of hypertension, some of which may include sleep apnea, drug-induced HTN, chronic kidney disease, diabetes, smoking, and physical inactivity. Physical assessment should include auscultation of heart tones and assessment of apical location to determine left ventricular hypertrophy. A comprehensive work-up should include lab studies, fasting lipid panel, thyroid-stimulating hormone, HgbA1c, and a basic metabolic panel. Urine for microalbuminuria may be indicated. Client education should include discussion of the importance of weight loss, smoking cessation, and reducing alcohol intake as well as directions for maintaining a heart-healthy low-sodium (Na) diet. Additional discussion should include maintaining an exercise program with a target of exercising most days of the week, with consideration of the client ' s limitations. Cardiac rehab should be instituted when indicated

The cholesterol component(s) considered most responsible for atherosclerotic plaque formation is (are) A. total cholesterol. B. low-density lipoprotein cholesterol. C. high-density lipoprotein cholesterol. D. phospholipids.

low-density lipoprotein cholesterol. Answer B The cholesterol component considered most responsible for atherosclerotic plaque formation is an elevated low-density lipoprotein cholesterol level.

Signs of right-sided heart failure include A. a low cardiac output. B. signs of fluid retention. C. dyspnea. D. elevated pulmonary venous pressure.

signs of fluid retention Answer B Signs of right-sided heart failure focus on fluid retention with edema and hepatic congestion, and depending on the extent of the disease, ascites will be evident. Signs of left-sided heart failure focus on a low cardiac output and an elevated pulmonary venous pressure, with dyspnea being the cardinal feature. It should be noted that heart failure may be systolic dysfunction, diastolic dysfunction or a combination of both. Treatment is directed contingent on the classification of heart failure (systolic or diastolic) and by the NY Heart Association Functional Class or the ACC /AHA stage.

Clinical findings associated with aortic regurgitation include A. pulsus paradoxus. B. waterhammer pulses. C. pulsus alternans. D. weak, thready pulses.

waterhammer pulses. Answer B Waterhammer pulses (Corrigan ' s pulses) are present in a client with aortic regurgitation or aortic insufficiency. The forceful, high-volume left ventricular ejection of blood into the aorta during systole is accompanied by a reflux of blood back into the left ventricle during diastole. Usually, the peripheral vascular resistance is low in aortic regurgitation, which maximizes the forward flow of blood into the periphery. As a result of this forceful movement of blood and subsequent backflow, the waveform of peripheral pulses is characterized by a rapid rise and collapse. Other findings resulting from this phenomenon include pistol-shot pulses (Duroziez ' murmur), which are audible on auscultation of the femoral artery; alternating flushing and paling of the nailbed capillaries (Quincke ' s capillary pulsation); and systolic head bobbing as the collapsed neck vessel fills rapidly (Musset ' s sign).


Kaugnay na mga set ng pag-aaral

Están haciendo muchas cosas- Look at the illustration and fill in the blanks using the present progressive to describe what each person is doing. Punctuation is already added at the end of the sentences.

View Set

Chapter 8 - Do It: Multiple Choice Quiz

View Set